\lecture{12}{2023-11-27}{} \subsection{Ordinal arithmetic} We define $+$, $\cdot $ and exponentiation for ordinals as follows: Fix an ordinal $\beta$. We recursively define \begin{IEEEeqnarray*}{rClClr} \beta &+& 0 &\coloneqq & \beta\\ \beta &+& (\alpha + 1)&\coloneqq &(\beta + \alpha) + 1,\\ \beta &+& \lambda &\coloneqq & \sup_{\alpha < \lambda} \beta + \alpha &~ ~\text{for limit ordinals $\lambda$} \end{IEEEeqnarray*} (Recall that $\alpha + 1 = \alpha \cup \{\alpha\}$ was already defined.) \begin{IEEEeqnarray*}{rClClr} \beta &\cdot& 0 &\coloneqq& 0,\\ \beta &\cdot& (\alpha+1) &\coloneqq & \beta\cdot \alpha + \beta,\\ \beta &\cdot& \lambda &\coloneqq &\sup_{\alpha < \lambda} \beta\cdot \alpha &~ ~\text{for limit ordinals $\lambda$} \end{IEEEeqnarray*} and \begin{IEEEeqnarray*}{rClr} \beta^0 &\coloneqq & 1,\\ \beta^{\alpha+1} &\coloneqq & \beta^{\alpha} \cdot \beta,\\ \beta^{\lambda} &\coloneqq & \sup_{\alpha < \lambda} \beta^{\alpha} &~ ~\text{for limit ordinals $\lambda$}. \end{IEEEeqnarray*} \gist{% \begin{example} \leavevmode \begin{itemize} \item $2+ 2 =4$, \item $196883 + 1 = 196884$, \item $1 + \omega = \sup_{n < \omega} 1 + n = \omega \neq \omega +1$, \item $2 \cdot \omega = \sup_{n < \omega} 2\cdot n = \omega$, \item $\omega \cdot 2 =\omega \cdot 1 + \omega = \omega + \omega$. \end{itemize} \end{example} }{} \gist{% \begin{warning} Cardinal arithmetic and ordinal arithmetic are very different! The symbols are the same, but usually we will distinguish between the two by the symbols used for variables (i.e.~$\alpha,\beta, \omega, \omega_1$ are viewed primarily as ordinals and $\kappa,\lambda, \aleph_\alpha$ as cardinals). \end{warning} We will very rarely use ordinal arithmetic. }{} \subsection{Cofinality} \begin{definition} Let $\alpha$, $\beta$ be ordinals. We say that $f\colon \alpha \to \beta$ is \vocab{cofinal} iff \gist{% for all $\xi < \beta$, there is some $\eta < \alpha$ such that $f(\eta) \ge \xi$.% }{% \[ \forall \xi < \beta.~\exists \eta < \alpha.~f(\eta) \ge \xi. \] } \end{definition} \gist{% \begin{remark} If $\beta$ is a limit ordinal, this is equivalent to \[ \forall \xi < \beta .~\exists \eta < \alpha.~f(\eta) > \xi. \] \end{remark} \begin{example} \begin{enumerate}[(a)] \item Look at $\omega + \omega$. \begin{IEEEeqnarray*}{rCl} f\colon \omega &\longrightarrow & \omega + \omega \\ n &\longmapsto & \omega + n \end{IEEEeqnarray*} is cofinal. \item Look at $\aleph_\omega$. Then \begin{IEEEeqnarray*}{rCl} f\colon \omega &\longrightarrow & \aleph_{\omega} \\ n &\longmapsto & \aleph_n \end{IEEEeqnarray*} is cofinal. \end{enumerate} \end{example} }{} \begin{definition} Let $\beta$ be an ordinal. The \vocab{cofinality} of $\beta$, denoted $\cf(\beta)$, is the least ordinal $\alpha$ such that there exists a cofinal $f\colon \alpha \to \beta$. \end{definition} \begin{example} \begin{itemize} \item $\cf(\aleph_\omega) = \omega$. In fact $\cf(\aleph_{\lambda}) \le \lambda$ for limit ordinals $\lambda \neq 0$ (consider $\alpha \mapsto \aleph_\alpha$). \item $\cf(\aleph_{\omega + \omega}) = \omega$. \end{itemize} \end{example} \begin{lemma} For any ordinal $\beta$, $\cf(\beta)$ is a cardinal. \end{lemma} \begin{proof} \gist{% Let $f\colon \alpha \to \beta$ be cofinal. Then $\tilde{f}\colon |\alpha| \to \beta$, the composition with $\alpha \leftrightarrow|\alpha|$ is cofinal as well and $|\alpha| \le \alpha$. }{Trivial.} \end{proof} \gist{% \begin{question} How does one imagine ordinals with cofinality $> \omega$? \end{question} No idea. }{} \begin{definition} An ordinal $\beta$ is \vocab{regular} iff $\cf(\beta) = \beta$. Otherwise $\beta$ is called \vocab{singular}. \end{definition} In particular, a regular ordinal is always a cardinal. \begin{lemma} Let $\beta$ be an ordinal Then $\cf(\beta)$ is a regular cardinal, i.e. \[\cf(\cf(\beta)) = \cf(\beta).\] \end{lemma} \begin{proof} Suppose not. Let $f\colon \cf(\beta) \to \beta$ be cofinal and $g\colon \cf(\cf(\beta)) \to \cf(\beta)$. Consider \begin{IEEEeqnarray*}{rCl} h\colon \cf(\cf(\beta)) &\longrightarrow & \beta \\ \eta &\longmapsto & \sup \{f(\xi): \xi \le g(\eta)\} < \beta. \end{IEEEeqnarray*} Clearly this is cofinal. \end{proof} \begin{warning} Note that in general, a composition of cofinal maps is not necessarily cofinal. \end{warning} \begin{theorem} Let $\kappa > \aleph_0$. Then $\kappa^+$ is regular. \end{theorem} \begin{proof} Suppose that $\cf(\kappa^+) < \kappa^+$. Then $\cf(\kappa^+) \le \kappa$, i.e.~there is a cofinal function $f\colon \kappa \to \kappa^+$. By the axiom of choice, there is a function $g$ with domain $\kappa$, such that $g(\eta)\colon \kappa \twoheadrightarrow f(\eta)$ is onto. Now define \begin{IEEEeqnarray*}{rCl} h\colon \kappa\times \kappa &\longrightarrow & \kappa^+ \\ (\eta, \xi) &\longmapsto & g(\eta)(\xi). \end{IEEEeqnarray*} Clearly this is surjective, but $|\kappa \times \kappa| < \kappa^+$, by \yaref{thm:hessenberg}. \end{proof} \gist{% \begin{itemize} \item $\aleph_0, \aleph_1, \aleph_2, \ldots$ are regular, \item $\aleph_\omega$ is singular, \item $\aleph_{\omega + 1}, \aleph_{\omega + 2}, \ldots$ are regular, \item $\aleph_{\omega + \omega}$ is singular, \item $\aleph_{\omega + \omega + 1}, \ldots$ are regular, \item $\aleph_{\omega + \omega + \omega}$ is singular, \item $\ldots$ \item $\aleph_{\omega_1}$ is singular, \item $\aleph_{\omega_1 + 1}, \ldots$ is regular, \item $\aleph_{\omega_2}$ is singular. \end{itemize} }{} \begin{question}[Hausdorff] Is there a regular limit cardinal? \end{question} Maybe. This is independent of $\ZFC$, cf.~\yaref{def:inaccessible}. \begin{theorem}[Hausdorff] \[ \aleph_{\alpha+1}^{\aleph_\beta} = \aleph_\alpha^{\aleph_\beta} \cdot \aleph_{\alpha+1}. \] \end{theorem} \begin{proof} \gist{% Recall that \begin{IEEEeqnarray*}{rCl} \aleph_{\alpha+1}^{\aleph_\beta} &=& |\leftindex^{\aleph_\beta} \aleph_{\alpha+1}|. \end{IEEEeqnarray*} \begin{itemize} \item First case: $\beta \ge \alpha+1$. Note that for all $\gamma \le \beta$ we have \[ \aleph_{\gamma}^{\aleph_\beta} \le \aleph_\beta^{\aleph_\beta} \le \left( 2^{\aleph_\beta} \right)^{\aleph_\beta} = 2^{\aleph_\beta \cdot \aleph_\beta} = 2^{\aleph_{\beta}} \le \aleph_\gamma^{\aleph_\beta}. \] So in this case $\aleph_\alpha^{\aleph_{\beta}} = 2^{\aleph_\beta}$ and $\aleph_{\alpha+1}^{\aleph_\beta} = 2^{\aleph_{\beta}}$. Thus \[ \aleph_{\alpha+1}^{\aleph_{\beta}} = 2^{\aleph_{\beta}} = \aleph_{\alpha}^{\aleph_\beta} = \aleph_{\alpha}^{\aleph_\beta} \cdot \aleph_{\alpha+1}. \] \item Second case: Suppose $\beta < \alpha+1$. By case hypothesis and because $\aleph_{\alpha+1}$ is regular, no $f\colon \aleph_{\beta} \to \aleph_{\alpha+1}$ is unbounded. So \[ \leftindex^{\aleph_{\beta}}\aleph_{\alpha+1} = \bigcup_{\xi < \aleph_{\alpha+1}} \leftindex^{\aleph_\beta} \xi \] for each $\xi < \aleph_{\alpha+1}$, $|\xi| \le \aleph_\alpha$, hence \[ |\leftindex^{\aleph_{\beta}}\xi| \le \aleph_\alpha^{\aleph_\beta} \] for each $\xi < \aleph_{\alpha+1}$. Therefore, \[\aleph_{\alpha+1}^{\aleph_\beta} \le \aleph_{\alpha+1} \cdot \aleph_{\alpha}^{\aleph_{\beta}} \le \aleph_{\alpha+1}^{\aleph_\beta}.\] \end{itemize} }{% $\ge$ is trivial. \begin{itemize} \item First case: $\beta \ge \alpha + 1$. Note that $\gamma \le \beta \implies \aleph_{\gamma}^{\aleph_\beta} = 2^{\aleph_\beta}$, so \[ \aleph_{\alpha+1}^{\aleph_\beta} \overset{\alpha+1 \le \beta}{=} 2^{\aleph_\beta} \overset{\alpha < \beta}{=} \aleph_\alpha^{\aleph_\beta} \le \aleph_\alpha^{\aleph_\beta} \cdot \aleph_{\alpha+1}. \] \item Second case: $\beta < \alpha+1$: \begin{itemize} \item $\aleph_{\alpha+1}$ is regular, so all $f\colon \aleph_\beta \to \aleph_{\alpha+1}$ are bounded. \item Thus $\leftindex^{\aleph_\beta}\aleph_{\alpha+1} = \bigcup_{\xi < \aleph_{\alpha+1}} \leftindex^{\aleph_\beta} \xi$ for all $\xi < \aleph_{\alpha+1}$. \item So $\aleph_{\alpha+1}^{\aleph_\beta} \le \aleph_{\alpha+1} \aleph_\alpha^{\aleph_\beta}$. \end{itemize} \end{itemize} } \end{proof}